Hand Tendons Flashcards

1
Q

A 30-year-old woman comes to the office because of pain and swelling of the right radial/distal forearm. The pain worsens with ulnar deviation of the wrist over a clasped thumb. Immobilization and a corticosteroid injection have failed, and surgical release is planned. Which of the following anatomic findings is most likely to have contributed to this patient’s condition?

A) Absent extensor pollicis brevis
B) Combined first and second compartments
C) Distal muscle belly on extensor pollicis brevis tendon
D) Multiple slips of abductor pollicis longus
E) Septum between abductor pollicis longus and extensor pollicis brevis

A

The correct response is Option E.

There is considerable variability in the anatomy of the first dorsal compartment of the wrist; this is particularly true when considering the presence or absence of a septum in the compartment. The significance of these variations is that they may predict failure of steroid injections to resolve de Quervain disease and must be considered when releasing the compartment surgically to ensure that a complete release of all tendons is performed.

While studies vary, based on methodology of imaging, cadaveric dissection, or surgical findings, there is consistency that septa are more common in patients presenting with de Quervain disease than in the general population.

Studies have shown no association between the number of APL slips and de Quervain disease.

2018

How well did you know this?
1
Not at all
2
3
4
5
Perfectly
2
Q

A 53-year-old woman comes to the office after undergoing fixation of a humerus fracture 17 months ago. Physical examination shows inability to extend the wrist, fingers, and thumb. This has been present since the time of injury, without any recovery of function. Tendon transfers are planned. Transfer of which of the following muscles is most appropriate for restoration of wrist extension?

A) Brachioradialis
B) Flexor carpi ulnaris
C) Flexor digitorum profundus
D) Palmaris longus
E) Pronator teres

A

The correct response is Option E.

The most appropriate muscle to transfer for restoration of wrist extension is the pronator teres.

The radial nerve can be injured as a result of humerus fracture and/or surgery as it crosses the spiral groove of the humerus. The resultant radial nerve palsy will cause inability to extend the wrist, fingers, and thumb.

Reinnervation of the muscle ideally should be completed within 12 to 18 months after injury to allow for recovery. In this patient, who has high radial nerve palsy after humerus fracture, the time following injury has been too long, so nerve repairs or nerve transfers are not a viable option, and tendon transfer is the procedure of choice.

Tendon transfer involves the use of a noncritical or expendable donor tendon to provide a missing function. The tendon to be transferred should have adequate strength and range of motion to provide the desired function. Ideally the tendon used should have synergistic action and allow for tenodesis to facilitate reeducation.

The pronator teres is a median nerve–innervated muscle that has adequate power and excursion to provide wrist extension. It is typically transferred to the extensor carpi radialis brevis (as opposed to the extensor carpi radialis longus) in these cases to provide for more centrally oriented wrist extension.

The brachioradialis is a radial nerve–innervated muscle and will not be functioning in this patient who has a high radial nerve palsy. In low radial nerve palsies, it can be used to restore thumb extension. The brachioradialis can also be used to restore finger or wrist extension, as well as finger or thumb flexion in the appropriate patient.

The flexor carpi ulnaris would have adequate power and excursion, but it is not synergistic and it would be difficult to learn to use a wrist flexor to power wrist extension, as it provides an opposite function. This is typically used to restore finger extension, as it would take advantage of the tenodesis effect.

The flexor digitorum profundus would be synergistic with wrist extension, but it does not have independent muscle bellies and its use would require sacrifice of important finger flexor activity.

The palmaris longus does not have sufficient power to provide for wrist extension. It can be used as a transfer for thumb extension.

2018

How well did you know this?
1
Not at all
2
3
4
5
Perfectly
3
Q

Which of the following Zone II four-strand flexor tendon repair configurations demonstrates the greatest overall fatigue strength and gap resistance in biomechanical testing?

A

The correct response is Option A.

Increasing the suture caliber has shown to increase the force in static testing and fatigue strength in dynamic testing. The use of 3-0 polyethylene terephthlate fiber suture increased the fatigue strength compared with 4-0 sutures by two to three times.

The placement of the suture dorsally has been shown to increase the strength of the repair by two to four times, and is more environmentally favored because of a decreased risk for interference with the synovial fluid.

It has been shown that locking loops improve force and gap resistance compared with grasping loops in flexor tendon repair. Statistically significant improvement was observed with the locking loop technique for ultimate and gap strength values using 2-0 core suture and ultimate strength values using 3-0 core suture.

2018

How well did you know this?
1
Not at all
2
3
4
5
Perfectly
4
Q

An 18-year-old female gymnast comes to the office because of the sudden onset of pain, swelling, and ecchymosis of the right ring finger that began 3 days ago while she was practicing hand-only climbing on a rock-wall. Physical examination shows tenderness over the palmar aspect of the finger at the proximal phalanx. Isolated flexion at the distal interphalangeal joint and flexion of the digit at the proximal interphalangeal joint while the remaining digits are held in hyperextension are intact. The patient can fully flex and extend the finger, and the proximal interphalangeal joint is stable to stress. X-ray study is shown. Which of the following is the most likely cause of the findings in this patient?

A) Avulsion of the flexor profundus tendon insertion
B) Dislocation of the proximal interphalangeal joint
C) Rupture of the A2 pulley
D) Stenosing tenosynovitis
E) Volar plate avulsion fracture

A

The correct response is Option C.

Stenosing tenosynovitis (trigger finger) typically results in pain over the metacarpophalangeal (MCP) joint associated with crepitation, clicking, or locking of the digit with altered motion during the flexion-extension arc.

Dislocation of the proximal interphalangeal (PIP) joint could account for the symptoms; however the imaging shows the joint to be congruent and the exam did not reveal any instability.

An injury originally identified in rock climbers, rupture of the A2 pulley has been increasingly recognized in other sports and activities. The forceful flexion of the flexor profundus and superficialis tendons with the PIP joint flexed 90 degrees and the distal interphalangeal (DIP) joint extended (the “crimp” position) placed loads exceeding the tolerance of the A2 across the palmar PIP joint, resulting in rupture of the underlying pulley. Treatment for an isolated pulley injury includes rest, ice, anti-inflammatory drugs, and external support through use of a ring splint.

Avulsion of the profundus tendon (jersey finger) typically presents with loss of flexion of the DIP joint. Pain may be present both at the avulsion site and over the retracted end of the tendon. This patient’s examination shows the profundus function to be intact.

Volar (palmar) plate avulsion injuries present with pain and swelling at the PIP joint, but the joint will often be painful to stress in hyperextension and potentially, if the injury extends dorsally into the collateral ligaments, it may exhibit instability. X-ray imaging often demonstrates a small bony avulsion fragment (absent in this patient) along the palmar PIP joint.

2018

How well did you know this?
1
Not at all
2
3
4
5
Perfectly
5
Q

A 62-year-old woman is evaluated for acute rupture of an extensor tendon after undergoing closed treatment of a nondisplaced distal radius fracture 6 months ago. The tendon most likely to be involved is located in which of the following extensor compartments?

A) First
B) Second
C) Third
D) Fourth
E) Fifth

A

The correct response is Option C.

Spontaneous rupture of the extensor pollicis longus (EPL) tendon is reported to occur in approximately 0.3 to 5% of nondisplaced or minimally displaced distal radius fractures, but it can also occur without trauma or in patients with inflammatory conditions such as rheumatoid arthritis. This is thought to arise from a loss of vascularity and atrophic changes in the compartment, and, because the tendon substance is usually degenerated, primary repair of the tendon is usually not possible. Tendon transfer using the extensor indicis proprius is the standard of care. Spontaneous rupture of other extensor tendons can occur in association with other conditions (e.g., rheumatoid arthritis), but would be exceedingly uncommon in the clinical scenario presented. The EPL passes through the third extensor compartment. Extensor tendon-compartment relationships include the following:

First - abductor pollicis longus, extensor pollicis brevis

Second - extensor carpi radialis longus, extensor carpi radialis brevis

Third - extensor pollicis longus

Fourth - extensor digitorum communis, extensor indicis proprius

Fifth - extensor digiti minimi

Sixth - extensor carpi ulnaris

2018

How well did you know this?
1
Not at all
2
3
4
5
Perfectly
6
Q

A 25-year-old man who is a graduate student comes to the office for evaluation of the right ring finger 4 weeks after sustaining an axial impact. A diagnosis of soft-tissue mallet finger is noted. Which of the following is the most appropriate treatment in this patient?

A) Arthrodesis
B) Orthosis
C) Pinning in extension
D) Tendon grafting
E) Terminal tendon repair

A

The correct response is Option B.

Most of these injuries even after a month will respond to splinting of the DIP joint in extension for 6 weeks. Any residual extension lag is largely an aesthetic concern and rarely will require further treatment. The operative treatment of soft-tissue mallet injury may lead to unacceptable complications while splinting may lead to skin irritation but little else.

The preferred treatment at 4 weeks is closed reduction and splinting. Surgical treatment may be considered for more chronic injury greater than 3 months. Pinning in extension may be appropriate for a small subgroup of patients with work-related inability to wear an orthosis (such as surgeons).

2017

How well did you know this?
1
Not at all
2
3
4
5
Perfectly
7
Q

A 60-year-old man sustained Zone II laceration to the left long finger, which severed both flexor tendons. A photograph is shown. Primary repair was performed with a 3-0 cruciate core suture within 2 weeks of injury. He is scheduled to undergo rehabilitation with active motion protocol. Compared with passive motion protocols, an active motion protocol is most likely to present which of the following risk factors and outcomes?

A) Equal risk of rupture and equal final range of motion
B) Higher risk of rupture and decreased final range of motion
C) Higher risk of rupture and increased final range of motion
D) Lower risk of rupture and decreased final range of motion
E) Lower risk of rupture and increased final range of motion

A

The correct response is Option C.

Many techniques for primary flexor tendon repair have been described. All methods involve a core suture to bear the load of the repair with or without an epitendinous suture. Multiple studies have shown that more strands crossing the repair site and/or larger diameter of each strand increases the strength of the repair. Two-strand repairs cannot withstand early active rehabilitation protocols; repairs of four strands or greater, including a cruciate repair as done in the patient above, can tolerate an early active motion rehabilitation protocol.

Trumble and Seiler’s studies compare passive motion protocols and active motion protocols to each other for risk of rupture and incidence of deceased range of motion at final measurement. Both studies identified active motion protocols incurred a higher occurrence of rupture but a lower incidence of decreased range of motion compared with the passive protocols.

2017

How well did you know this?
1
Not at all
2
3
4
5
Perfectly
8
Q

A 54-year-old right-hand–dominant man comes to the office because of a 1-year inability to fully extend the right thumb after sustaining a laceration. Medical history includes no abnormalities.The patient reports being unable to grasp large objects. Hand and wrist x-ray studies show no abnormalities. An extensor pollicis longus (EPL) tendon injury is suspected. Exploration is planned. Which of the following additional interventions is the most appropriate next step in management?

A) Primary four-strand repair of the EPL tendon
B) Repair of the EPL tendon with a palmaris longus tendon interposition graft
C) Tenorrhaphy of the extensor pollicis brevis tendon and EPL tendon
D) Transfer of the extensor indicis proprius to the EPL tendon
E) Transfer of the palmaris longus tendon to the EPL tendon

A

The correct response is Option D.

Extensor indicis proprius (EIP) transfer is the most common procedure for the treatment of chronic ruptures of the extensor pollicis longus (EPL) tendon. The EPL tendon is prone to rupture from synovitis and friction at Listers tubercle. Since these are identified months after the original injury, primary repair is not possible secondary to retraction of the tendon and/or atrophy of the tendon ends. The EIP is the preferred tendon for the transfer because it has an appropriate direction and excursion compared with the EPL. This tendon transfer has demonstrated excellent outcomes in previous studies. Tenorrhaphy of the EPL to the extensor pollicis brevis tendon would not allow full thumb extension.

Given the chronicity of this injury, sufficient myostatic contracture has occurred to render tendon interposition grafting inferior to EIP tendon transfer.

2017

How well did you know this?
1
Not at all
2
3
4
5
Perfectly
9
Q

A 22-year-old man comes to the office because of injury to the right index finger flexor tendons in Zone II. During open repair, which of the following flexor tendon pulleys arise from volar plates?

A) A1, A2, and A3
B) A1, A3, and A4
C) A1, A3, and A5
D) A2 and A4
E) A4 and A5

A

The correct response is Option C.

The flexor tendons are bound within a fibro-osseous sheath, with pulleys essential to prevent bowstringing (and consequent poor force transfer). The pulley system includes both annual and cruciate pulleys. Pulleys A2 and A4 attach to bone. Pulleys A1, A3, and A5 are attached to the volar plates at their respective joints. The strength of pulleys, in order from strongest to weakest, are the A2, A1, and A4 pulleys. The pulleys attached to bone have a higher breaking strength compared with those attached to the volar plates.

2017

How well did you know this?
1
Not at all
2
3
4
5
Perfectly
10
Q

A 53-year-old woman comes to the office because of pain of the dorsum of the left wrist and thumb for the past 3 months. The patient reports that pain occurs with activity. Physical examination shows pain is increased with passive wrist ulnar deviation with the thumb held in the palm and during resisted extension of the thumb metacarpophalangeal (MCP) joint. Axial loading of the thumb does not reproduce pain. An x-ray study is shown. Which of the following is the most likely diagnosis in this patient?

A) Basal joint arthritis
B) de Quervain tenosynovitis
C) Intersection syndrome
D) Scaphotrapezial arthritis
E) Stenosing tenosynovitis

A

The correct response is Option B.

Pain in the dorsal radial aspect of the wrist can be caused by a variety of conditions; a thorough history and physical examination are key to elucidating the correct diagnosis. In the scenario presented, the patient has a positive Finkelstein test along with reproduction of the pain with resistance to the extensor pollicis brevis muscle. These are classic findings of de Quervain tenosynovitis. The patient’s physical examination points to the first dorsal compartment of the wrist as a source of pathology rather than basal joint or scaphotrapezial arthritis. Radiographs have not been shown to correlate with symptomatology in basilar joint arthritis. Intersection syndrome is a tendinopathy between the intersection of the tendons of the first and second dorsal compartments. The pain of intersection syndrome is generally found more proximally in the forearm and is also increased with resisted wrist extension. Digital flexor tenosynovitis or trigger finger is an inflammatory tendinopathy of the flexor pollicis longus tendon and pain is generally reproduced with resisted thumb flexion.

2017

How well did you know this?
1
Not at all
2
3
4
5
Perfectly
11
Q

A 50-year-old man who is a biathlete comes to the office because of weakness and pain when gripping or pinching with the left hand. Medical history includes a sprain to the left thumb with forced radial abduction 1 year ago. Physical examination shows a difference in stability of the right thumb and the left thumb during stress testing. Photographs are shown. Which of the following is the most appropriate treatment for this patient’s metacarpophalangeal (MCP) joint injury?

A) Direct repair of the collateral ligament
B) Graft reconstruction of the collateral ligament
C) Occupational therapy for strengthening of the adductor muscle
D) Placement of a short opponens splint for 6 weeks
E) Transfer of the extensor indicis proprius tendon to the adductor insertion

A

The correct response is Option B.

Injuries to the ulnar collateral ligament (UCL) of the thumb metacarpophalangeal joint (skier’s thumb Injury) can be successfully treated with 4 weeks of immobilization provided that the injury results in either no instability, or limited (<30 to 35-degree laxity under load, or <10 to 15-degree difference from the contralateral UCL under load) instability. The photographs provided show 40 to 45 degrees of laxity, which differs substantially from the contralateral (<10 degrees) thumb. Treatment of this injury requires operative intervention. Incomplete injuries, or injuries with only mild symptomatic laxity, may benefit from occupational therapy. The adductor muscle provides a stabilizing force across the metacarpophalangeal joint and has been a target for treating mild injuries that result in some degree of instability. This would be insufficient for treating this complete rupture. In a similar fashion to strengthening the adductor muscle, addition of another force directed at providing ulnar adduction at the metacarpophalangeal joint has been proposed. As in the case of adductor strengthening, however, this would be inadequate to treat the complete rupture apparent in the clinical photos. When encountered acutely, unstable, complete ruptures of the ulnar collateral ligament of the thumb metacarpophalangeal joint are optimally treated with operative repair. Four to six weeks following the injury, direct repair may be difficult if not impossible. In this patient, the presentation for treatment occurs one year after the initial injury, making repair of the UCL extremely unlikely. On operative exploration in this patient, only a shortened stump of UCL remained attached to the metacarpal head. Chronic unstable injuries of the thumb metacarpophalangeal (MCP) joint ulnar collateral ligament (UCL) may be symptomatic via weakened grip and pinch as well as pain. Treatment in this setting will be dictated by patient needs and by the status of the joint. Patients requiring mobility and lacking arthritic degeneration at the MCP joint are candidates for UCL reconstruction. Reconstruction will typically be accomplished by use of a tendon graft, either palmaris or plantaris, placed through bone tunnels and secured through one of multiple methods (interference screws, periosteal sutures, bone anchors). Given this patient’s presentation one year out from the initial injury, reconstruction is the best option.

How well did you know this?
1
Not at all
2
3
4
5
Perfectly
12
Q

A 60-year-old woman presents with weakness and inability to fully extend the right dominant thumb at the interphalangeal joint. History includes a Colles fracture of the right wrist 6 months ago. Management of the fracture included cast immobilization. On physical examination, the patient’s thumb is at 30 degrees of flexion. Finger metacarpophalangeal joint active extension is normal. The patient cannot extend or lift the thumb with her hand flattened on a table. With the thumb adducted, she can extend it to neutral. All thumb joints are supple and have full range of passive motion. Which of the following is the most likely cause of this patient’s inability to extend the thumb?

A) Intersection syndrome
B) Radial nerve palsy
C) Rupture of the extensor pollicis longus
D) Saddle deformity of the basal joint
E) Trigger thumb with locking

A

The correct response is Option C.

Extensor pollicis longus (EPL) rupture is most commonly caused by late effects of distal radius fractures. Devascularization is the most likely cause leading to attritional rupture. Rupture can present 2 weeks to 11 months after fracture; the average is 7 weeks. This patient can extend her thumb when it is adducted because of connections of the intrinsics with the dorsal apparatus. Tendon transfer of the extensor indicis proprius to distal EPL stump is the first-line treatment.

Other causes of EPL rupture include synovitis from rheumatoid arthritis and lupus causing friction at Lister’s tubercle, steroid injections, excessive abnormal wrist motion, bony spurs following distal radius and scaphoid fractures, scaphoid nonunion, misplaced external fixator pin, and subluxation of the distal ulna.

A trigger thumb would more likely have pain and tenderness on physical examination along the volar flexor sheath. A locked thumb would not have passive extensibility or active extension with adduction.

Saddle deformity is seen on physical examination in advanced osteoarthritis of the basal joint. Interphalangeal joint flexion of the thumb is caused by zigzag longitudinal collapse with hyperextension of the metacarpophalangeal joint.

Intersection syndrome is characterized by pain and tenderness along the radial side of the forearm where the abductor pollicis longus and extensor pollicis brevis muscles intersect.

Radial nerve palsy would not only affect the thumb interphalangeal joint.

2016

How well did you know this?
1
Not at all
2
3
4
5
Perfectly
13
Q

A 19-year-old college baseball player comes for evaluation 4 weeks after he jammed and dislocated the long finger of his dominant right hand while sliding into home plate. His coach reduced the dislocation on the field. The patient says he has noticed increasing pain at the site of the injury in the past two days. Physical examination shows edema of the proximal interphalangeal (PIP) joint of the long finger. Lateral stress shows angulation of 30 degrees. X-ray study shows a congruous joint with radial side widening of 1 mm. Which of the following is the most appropriate management?

A) Buddy taping of the long finger to the index finger
B) Immobilization in a dorsal extension block splint at 30 degrees
C) Immobilization with a volar short arm splint
D) Kirschner wire fixation of the PIP joint for 3 weeks
E) Open repair of the radial collateral ligament

A

The correct response is Option E.

The goal of treatment is to restore normal finger function; prevent pain, stiffness, and traumatic arthritis; and to restore activities of daily living. This patient has a complete tear of the radial collateral ligament of the proximal interphalangeal (PIP) joint of the long finger. Widening of the joint space indicates probable interposition of ligament fibers. Angulation greater than 20 degrees is associated with poor prognosis. Complete tears with subluxation and interposition require surgical repair.Most collateral ligament ruptures occur at the proximal attachment to the middle phalanx.

Partial tears can be treated with nonsurgical and conservative methods. Splinting, buddy taping, extension block placement, and temporary fixation with a Kirschner wire would be incorrect.

2016

How well did you know this?
1
Not at all
2
3
4
5
Perfectly
14
Q

A 24-year-old woman comes for evaluation 6 days after sustaining a jamming injury to the long finger of the left hand with resultant central slip disruption and acute boutonnière injury. X-ray studies are negative for fracture or dislocation. Treatment with splint immobilization is planned. Which of the following is the most appropriate position of the finger for application of the splint?

A) Distal interphalangeal (DIP) joint extended, proximal interphalangeal (PIP) joint extended
B) DIP extended, PIP free
C) DIP flexed, PIP flexed
D) DIP flexed, PIP free
E) DIP free, PIP extended

A

The correct response is Option E.

The most appropriate position for splint immobilization of an injury leading to a boutonnière deformity is with the distal interphalangeal (DIP) joint free and the proximal interphalangeal (PIP) joint extended.

The patient sustained a central slip disruption of the long finger of the left hand, resulting in an acute boutonnière deformity. Patients with boutonnière deformity have flexion at the PIP joint and hyperextension at the DIP joint. There is tearing of the extensor tendon from its insertion at the base of the middle phalanx, resulting in decreased ability to extend the finger at the PIP joint. As a result, the lateral bands fall volar to the axis of rotation at the PIP joint, changing their force from extension to flexion at the PIP joint. The extensor force of the lateral bands is then directed toward the terminal tendon at the DIP joint, resulting in hyperextension of the DIP joint.

Splint immobilization of the PIP joint in extension and the DIP joint free to flex permits healing of the central slip back to the middle phalanx. Flexion of the DIP joint through movement will tend to cause the lateral bands to migrate dorsally again, reversing the effect of the boutonnière deformity. When the lateral bands move dorsal to the axis of rotation at the PIP joint, their extensor force is restored and the DIP hyperextension resolves.

Splint immobilization of both joints in extension will not encourage the dorsal migration of the lateral bands. Splint immobilization with the DIP joint in extension and the PIP joint free is recommended in zone 1 extensor tendon injuries (i.e., mallet finger).

2016

How well did you know this?
1
Not at all
2
3
4
5
Perfectly
15
Q

A 47-year-old man undergoes repair of a laceration to the extensor tendon of the long finger at the dorsum of the left hand. He wishes to restore function of the hand as quickly as possible. Which of the following is the most appropriate course of splint immobilization?

A) Continuous extension splint immobilization for 4 weeks
B) Continuous extension splint immobilization for 6 weeks
C) Dynamic extension splint immobilization for 6 weeks
D) Relative motion extension splint immobilization for 6 weeks
E) No immobilization

A

The correct response is Option D.

The most appropriate treatment is relative motion extension splint immobilization for 6 weeks.

The relative motion extension splint allows immediate controlled active motion. Placing the injured tendon in 15 to 20 degrees less motion than the adjacent tendons results in significantly less force. A splint is fashioned placing the repaired extensor tendon of the long finger in 15 to 20 degrees more metacarpophalangeal extension than the neighboring extensor tendons. Full interphalangeal joint range of motion is permitted. The finger splint is worn with a wrist component for the first 3 weeks (which may not be necessary). The finger component is used for only 3 additional weeks. This allows for earlier recovery of motion and return to work.

Without immobilization, the repair is at increased risk for rupture and failure. Extensor tendon injuries have typically been treated with continuous extension splint immobilization for 4 to 6 weeks. Although this would be a reasonable option, use of the relative motion extension splint allows immediate movement and decreases the stiffness that may result from immobilization. This may be preferred in children or noncompliant patients.

Dynamic extension splinting is a possibility and may be useful in patients who have an extensor pollicis longus injury or in patients in whom all six finger extensors are severed. Relative motion splinting is not an option in these cases.

2016

How well did you know this?
1
Not at all
2
3
4
5
Perfectly
16
Q

A 42-year-old woman comes to the emergency department after sustaining a deep laceration to the dorsal aspect of the right forearm with broken glass. She reports pain on attempted extension of the middle and ring fingers of the right hand. On physical examination, the patient cannot extend her fingers beyond the neutral position with her right hand held flat on a table. She is able to extend all digits completely at the interphalangeal joints in all positions of the hand. Which of the following is the most likely cause of these findings?

A) Extrinsic extensor tendons are cut completely with intact intrinsic muscles
B) Extrinsic extensor tendons can extend the digits despite the laceration injury
C) Juncturae tendinum are extending the digits
D) The patient has an accessory extensor tendon
E) The patient has partial extensor tendon lacerations only

A

The correct response is Option A.

Extension of the interphalangeal joints (IP) and extension of the metacarpophalangeal joints (MCP) to the neutral position after an injury to the forearm extensor tendons is possible through the action of the lumbricals and dorsal interossei (intrinsic hand muscles). The extrinsic extensor tendons alone are responsible for extending the metacarpophalnageal joints beyond neutral.

The role of the juncturae tendinum is to limit the independent extension of the ulnar three digits. The juncturae tendinum will allow for some finger extension with an isolated single tendon injury at the wrist or dorsal hand but would not maintain extension in all fingers.

No meaningful accessory extensor tendon exists.

A patient with partial extrinsic tendon lacerations may present with pain on attempted extension but extension beyond neutral would be intact.

2016

How well did you know this?
1
Not at all
2
3
4
5
Perfectly
17
Q

A 35-year-old woman is evaluated 2 months after repair of volar lacerations to the dominant ring and long fingers at Zone III in both digits. Physical examination shows both fingers have no active flexion. There is normal passive motion and normal sensation at the fingertips of the affected digits. Surgical exploration shows transsection of the tendons at both levels and 2-cm segmental tendon loss, but no tendon sheath scarring. Which of the following is the most appropriate management?

A) One-stage tendon grafting
B) Primary tendon repair
C) Primary tendon repair with z-lengthening
D) Superficialis to profundus tendon transfer
E) Two-stage tendon grafting

A

The correct response is Option A.

Several conditions must be met for single-stage tendon grafting to be successful. These include a hand and finger that have good passive motion, a well-healed wound with minimal scarring, and a digit that has intact nerves and arteries. This is a Boyes grade 1 injury.

If the grade of injury is greater than 1, two-stage grafting should be considered with implantation of a silicone rod and additional treatment to manage the other conditions to increase motion and function, such as joint release or reconstruction for loss of motion, nerve repair/reconstruction, and pulley reconstruction.

A primary repair is not going to be possible in this instance. Two-stage tendon grafting should be considered but a single stage graft can have excellent results with only one procedure. A tendon transfer is not appropriate for this situation.

2016

How well did you know this?
1
Not at all
2
3
4
5
Perfectly
18
Q

A 35-year-old man has clawing of all four fingers of the right hand 2 years after repair of a forearm laceration that injured the median and ulnar nerves. Photographs are shown. Both nerves were repaired shortly after the injury. All fingers have full passive range of motion. An extensor carpi radialis longus transfer is planned to correct the clawing of all four fingers. Which of the following donor sites is most likely to provide sufficient tendon graft for this procedure?

A) Abductor digiti quinti
B) Extensor digitorum longus
C) Flexor digitorum superficialis of the long finger
D) Palmaris brevis
E) Pronator teres

A

The correct response is Option B.

The Brand transfer uses the extensor carpi radialis longus or brevis as a donor motor to correct clawing of the fingers. It can be used to correct ulnar (ring and small finger) clawing or clawing of all four fingers. In either case, a tendon graft is needed to bridge the gap between the native distal limit of the extensor carpi radialis longus or brevis (on the index or long finger metacarpal base, respectively) to the transfer insertion on the lateral band at the proximal phalanx level. The transfer can be passed through the interosseous membrane in the forearm and then through the carpal tunnel or the intermetacarpal spaces in the hand. The transfer must pass volar to the deep transverse metacarpal ligament to have the correct vector of pull.

The extensor digitorum longus provides four tendon slips distally, each with its own paratenon, but has one tendon coming off the muscle proximally. A photograph is shown. There is minimal donor site morbidity in the foot due to the retained function of the extensor digitorum brevis.

Palmaris brevus has no tendon and cannot be used as a graft donor.

Abductor digiti quinti can be used as a donor muscle for thumb opposition transfer, most commonly in children. It has a very short tendon and cannot be used for anti-claw transfers. In addition, due to this patient’s injury, the abductor digiti quinti is likely denervated and would not be functional for a transfer.

Flexor digitorum superficialis (FDS) of the long finger can be used for anti-claw transfers. The tendon can be split along the plane of Camper’s chiasm to create two distal slips for insertion. It is well suited as an anti-claw transfer in patients with isolated ulnar nerve palsy. The FDS tendon cannot be split into four slips for insertion, as would be needed in this patient, and the power of one FDS muscle is insufficient to correct clawing in four fingers. In addition, the FDS resides superficial to the median nerve. In a patient who sustained a forearm laceration with injury to the median nerve, the overlying FDS is also likely to have been injured.

Pronator teres is the most common donor motor to restore wrist extension in patients with radial nerve palsy. It has a very short tendon and is not used for anti-claw transfers.

2016

How well did you know this?
1
Not at all
2
3
4
5
Perfectly
19
Q

A 38-year-old woman comes for evaluation 7 weeks after undergoing a Zone II flexor tendon repair of the left long finger. She reports feeling a “pop” at home and is now unable to flex the finger at the proximal or distal interphalangeal joints. On examination today, the finger is swollen with moderate stiffness. Flexor tendon rupture is suspected. During operative exploration, ruptures of the flexor digitorum superficialis (FDS) and flexor digitorum profundus tendons are noted. There is a 1.5-cm gap of the profundus tendon and scarring at the A2 pulley. Which of the following is the most appropriate next step in management?

A) Excision of the flexor tendons with implantation of a passive silicone rod prosthesis
B) Fractional lengthening of the tendon in the forearm and revision of primary repair
C) Single-stage reconstruction with implantation of an active silicone rod prosthesis
D) Single-stage tendon reconstruction with palmaris graft
E) Tendon transfer from the ring finger FDS

A

The correct response is Option A.

Flexor tendon reconstruction in Zone II was originally described by Bassett and Carroll in 1963 and refined by Hunter in 1971. In the first stage, a Dacron-reinforced silicone rod is implanted after excision of the native tendons. It is secured distally to the flexor digitorum profundus stump or directly to the distal phalanx. Proximally, the rod is placed adjacent to the motor tendon but not secured. At this time, pulley reconstruction with tendon or retinacular grafts can be performed as indicated. This allows formation of a pseudosheath around the rod. Once the soft tissue has healed and the patient has regained maximum passive range of motion through therapy, the second-stage tendon grafting is performed. The most commonly reported time frame is 3 months but depends on soft-tissue stability.

The most important management decision is to determine if primary repair is possible. If not, one must then decide between single-stage and two-stage tendon reconstruction. The criteria for single-stage reconstruction include a finger with adequate passive motion of all joints, soft tissues with minimal scarring, functional tendon sheath and pulley system, a neurovascularly intact digit, and a compliant patient. If these criteria are not met, the patient should be treated with a staged reconstruction.

In this patient, primary repair is not likely with a gap >1 cm 7 weeks after the initial repair. Myostatic contraction of the proximal stump would be expected. In addition, this patient’s digit has significant internal scarring and poor passive motion— all factors that preclude a single-stage repair. Fractional lengthening might be considered to allow primary repair if the tendon sheath was better quality.

If a single-stage repair were indicated, one may consider a flexor digitorum superficialis transfer from an adjacent digit as a motor for the transfer. This requires only one tendon anastomosis, and studies have shown decreased adhesion formation with intrasynovial tendon grafts. However, tensioning of the transfer can be more difficult than traditional tendon grafting.

Patients who are unable to tolerate a second-stage procedure can be considered for the implantation of an active silicone rod prosthesis. This device is designed to have both a distal anastomosis to the bone and a proximal anastomosis to the motor muscle-tendon unit with integrated sutures or a loop. This would require meeting the criteria for single-stage reconstruction. Active implants were originally designed for use in two-stage reconstruction, but no studies exist that compare active with passive silicone rods or show any benefit versus a passive implant in a staged reconstruction.

2016

How well did you know this?
1
Not at all
2
3
4
5
Perfectly
20
Q

A 28-year-old woman is brought to the emergency department after sustaining an injury to the arm during a motor vehicle collision. A photograph is shown. Physical examination shows inability to extend the index or long fingers, and a rent in the dorsal wrist capsule. Which of the following extensor zones is most likely involved?

A) II
B) III
C) IV
D) V
E) VI

A

The correct response is Option E.

The patient described sustained a dorsal wrist injury involving extensors of the wrist and fingers, with the injury at the base of the hand and over the carpus. Typically this is considered zones VI and VII. The other zones represent the following areas with some common conditions listed for each zone. Knowing and reporting zones of injury are helpful not just for descriptive purposes, but also for communicating with other doctors and therapists. Common therapy protocols are based on which zone or zones are involved in the injury.

2016

How well did you know this?
1
Not at all
2
3
4
5
Perfectly
21
Q

A 24-year-old male figure skater comes to the emergency department because of dorsal metacarpophalangeal dislocation of the right thumb. The emergency department staff is unable to reduce the dislocation. Which of the following structures is most likely preventing reduction in this patient’s injury?

A) Extensor pollicis longus tendon
B) Flexor pollicis brevis tendon
C) Opponens pollicis
D) Sesamoid bone
E) Ulnar neurovascular bundle

A

The correct response is Option D.

The thumb metacarpophalangeal (MCP) sesamoid bone(s) may be associated with an irreducible dorsal dislocation.

Dorsal dislocation of the thumb at the MCP joint typically occurs with forced hyperextension with resultant volar plate and collateral ligament rupture. Irreducibility usually occurs through interposition of the volar plate in the joint. Extensor expansion interposition, collateral ligaments, bony fragments, sesamoids, and flexor pollicis longus entrapment have also been associated with irreducibility. In these circumstances, open reduction is often necessary.

The ulnar neurovascular bundle, extensor pollicis longus, opponens pollicis, and flexor pollicis brevis are not typically associated with an irreducible dislocation.

2016

How well did you know this?
1
Not at all
2
3
4
5
Perfectly
22
Q

A 34-year-old machinist undergoes repair of the flexor tendon of the index finger as the result of a work-related injury. Which of the following is the main rationale for performing early motion exercises after surgical repair?

A) Decrease adhesions
B) Decrease postoperative pain
C) Improve strength of repair
D) Increase synovial fluid flow
E) Prevent rupture

A

The correct response is Option A.

The main rationale for performing early motion exercises is to decrease adhesion formation. During the early phases of tendon healing, large amounts of collagen are deposited and form early scarring. Although this scarring is crucial for the healing of the repaired tendon, scarring will also occur in the tendon sheath and, if allowed to progress, can lead to stiffness in the involved digit. Early passive- and active-motion protocols assist in breaking apart early scarring of the tendon to the surrounding sheath.

Early motion has not shown to increase the final strength of the repair and is more likely to cause rupture. Although synovial fluid flow might be increased with tendon excursion, it does not improve outcomes. Postoperative pain is likely increased with therapy.

2015

How well did you know this?
1
Not at all
2
3
4
5
Perfectly
23
Q

A 21-year-old man comes to the office after injuring the right long finger while playing football. On physical examination, the patient is unable to flex the distal phalanx. Hollowness is noted along the volar aspect of the finger, with pain in the palm upon palpation. X-ray studies show no abnormalities. Which of the following is the latest at which primary repair is expected?

A) 1 day
B) 1 week
C) 1 month
D) 2 months
E) 3 months

A

The correct response is Option B.

Rupture of the flexor digitorum profundus (FDP) tendon from its distal attachment is commonly known as jersey finger. The injury is often overlooked by players and trainers and misdiagnosed as a sprained finger, but it requires more urgent management than these minor injuries. Jersey finger occurs when a flexed distal interphalangeal (DIP) joint is suddenly and forcefully hyperextended, leading to rupture of the FDP tendon at its insertion on the distal phalanx.

FDP injuries can be classified based upon the degree of tendon retraction, as described in Leddy and Packer’s grading scheme:

Type I injuries involve retraction of the profundus tendon all the way to the palm, with associated injuries to the vincula longus and vinculum brevis. Injuries to the vincula disrupt the blood supply to the tendon, necessitating surgical repair within 7 days to avoid necrosis of the tendon and a permanent contracture deformity.

Type II injuries involve retraction of the tendon to the proximal interphalangeal (PIP) joint. The tendon stump is held in place by the vincula longus, which are often intact. An avulsion fracture sometimes occurs with type II injuries, and often becomes trapped in the A2 pulley. Without an observable bony fragment on x-ray study, it is impossible to determine the degree of retraction; thus, all type II injuries should be surgically repaired within 7 days.

Type III injuries involve a large avulsion fragment that is often intraarticular. The bony fragment prevents retraction past the A4 pulley and holds the tendon in near-anatomic position, obviating the need for urgent repair. Type III injuries are amenable to repair within 2 to 3 months.

Type IV injuries are type III lesions with the addition of an avulsion of the FDP tendon from the fracture fragment. Type IV injuries are rare but require urgent repair because of the disruption to the tendon’s blood supply.

2015

How well did you know this?
1
Not at all
2
3
4
5
Perfectly
24
Q

A 45-year-old man has a mass in the left volar forearm. An MRI is shown. Oncologic resection will involve removing all muscles of the anterior forearm compartment. Which of the following techniques is most appropriate to reconstruct finger flexion postoperatively?

A) Flexor digitorum profundus to superficialis transfer
B) Flexor pronator slide
C) Free gracilis innervated by a median nerve branch
D) Oberlin-Mackinnon nerve transfer
E) Pedicled latissimus dorsi muscle flap transfer

A

The correct response is Option C.

A free gracilis muscle would provide a good strength and excursion match to the native finger flexors it would replace. Vascular and nerve connections could be performed in the distal upper arm, outside of the zone of resection. While individual finger flexion would not be restored, the patient would be able to make a composite fist after this surgery.

The flexor digitorum profundus to superficialis transfer is used to treat flexor spastic contracture, typically with palmar hygiene issues, in a patient who still has some voluntary motor control. It requires the presence of flexor digitorum profundus and superficialis muscles, both of which would be removed as part of the anterior forearm compartment muscles.

The Oberlin-Mackinnon nerve transfer transfers branches of the median nerve and ulnar nerve to the brachialis and biceps muscles, respectively. It is used to restore elbow flexion and would not provide finger flexion.

A flexor pronator slide detaches the muscles originating from the medial epicondyle of the humerus and advances them distally. It is used to treat contractures of the fingers, often in patients with mild to moderate Volkmann contracture. In this patient, the flexor muscles have been resected, so this procedure is not possible.

A latissimus dorsi muscle provides broad soft-tissue coverage for wounds and can also provide a strong flexion force. It cannot reach beyond the distal elbow. It can restore elbow flexion but cannot be used as a pedicled transfer to restore finger flexion.

2015

How well did you know this?
1
Not at all
2
3
4
5
Perfectly
25
Q

A 35-year-old woman who is an artist is evaluated because of Boutonnière deformity with a flexed proximal interphalangeal (PIP) joint and an extended distal joint. She reports sustaining a laceration from a paint knife to the nondominant midline dorsal PIP joint 6 months ago. She did not seek medical attention at the time of injury. This patient’s deformity is a result of attenuation of which of the following structures?

A) Oblique retinacular
B) Sagittal bands
C) Terminal tendon
D) Triangular ligament
E) Volar plate

A

The correct response is Option D.

A Boutonnière deformity occurs with injury to the central tendon and injury or attenuation of the triangular ligament. The lateral bands migrate volarly to become proximal interphalangeal (PIP) joint flexors, and their action is on the distal interphalangeal (DIP) joint, extending it. The triangular ligaments are bound by the lateral bands, central slip, and terminal tendon.

The sagittal bands of the metacarpophalangeal (MCP) joint originate from the volar plate and anchor the extensor mechanism. The oblique retinacular ligament originates from the volar lateral crest of the proximal phalanx and inserts into the terminal tendon. The volar plates stabilize the MCP and PIP joints. The terminal tendon is the convergence to the lateral bands at the dorsum of the middle phalanx inserting on the distal phalanx.

2015

26
Q

A 39-year-old man is referred to the office 4 months after repair of a zone II flexor tendon involving both the flexor digitorum superficialis and flexor digitorum profundus tendons to the right long finger. He still has poor range of motion of his long finger. Physical examination shows 45 degrees of active range of motion at the proximal interphalangeal (PIP) joint and 25 degrees of active range of motion at the distal interphalangeal (DIP) joint. Active and passive ranges of motion are equal. Which of the following is the most appropriate next step in management?

A) Continued hand therapy to improve passive range of motion
B) Flexor tenolysis
C) PIP arthrodesis
D) Two-stage flexor tendon reconstruction
E) Observation to allow for scar remodeling

A

The correct response is Option A.

This patient has adhesions after flexor tendon injury and repair. In this scenario, the recommended course of action is to continue hand therapy to improve passive range of motion. A successful functional outcome following tendon injury depends on supple joints with full passive range of motion and tendon gliding. The ultimate goal would be to perform flexor tenolysis; however, the indications are clear that the patient must have minimal soft-tissue edema, minimal scarring, and full or near-full passive range of motion.

Active tendon range of motion depends on the flexor digitorum superficialis and flexor digitorum profundus gliding within the flexor tendon sheath. Flexor tendon adhesions are a potential complication any time the flexor tendon sheath is violated, as a result of either surgery or trauma. The literature shows a reoperation rate of 6% after flexor tendon repair and an adhesion rate of 4%.

This patient is 4 months post-surgery and reports compliance with supervised hand therapy. Despite this, he has poor active and passive range of motion. Although most authors recommend waiting at least 3 to 6 months before attempting tenolysis, this patient is unlikely to improve with observation alone.

Tenolysis is a technically demanding procedure, and all patients must be counseled preoperatively that complications such as neurovascular injury, injury to the pulley system, and tendon rupture are possible outcomes. In cases of tendon rupture or patients requiring pulley reconstruction at the time of tenolysis, two-stage tendon reconstruction with implantation of a silicone rod is indicated. However, this would be a salvage procedure only and not first-line treatment.

Proximal interphalangeal (PIP) joint arthrodesis would be limited to a salvage procedure in patients who are unable to undergo tendon repair or reconstruction.

2015

27
Q

Following central slip injury, volar subluxation of the lateral bands can lead to which of the following deformities?

A) Boutonnière
B) Lumbrical plus
C) Mallet
D) Quadriga
E) Swan-neck

A

The correct response is Option A.

The triangular ligament stabilizes the lateral bands dorsally, thereby preventing volar subluxation of the lateral bands to the proximal interphalangeal (PIP) joint rotation of axis, and the boutonnière deformity.

The swan-neck deformity occurs when the lateral bands sublux dorsal to the PIP joint rotation of axis. This is prevented by the transverse retinacular ligament, which acts to prevent dorsal migration of the lateral bands at the PIP joint.

Neither the mallet, quadriga, or lumbrical plus deformities are caused by volar subluxation of the lateral bands.

2015

28
Q

A 16-year-old boy is brought to the office after “jamming” the right long finger of the dominant hand while playing football. Upon active extension, the patient exhibits an extension lag of 40 degrees at the proximal interphalangeal (PIP) joint, and hyperextension at the distal interphalangeal (DIP) joint. Which of the following is the most likely diagnosis?

A) Central slip disruption
B) Flexor digitorum profundus avulsion
C) PIP volar plate tear
D) Sagittal band rupture
E) Swan neck deformity

A

The correct response is Option A.

This is the basis of the Elson test for central slip disruption of the extensor mechanism of the finger. When the proximal interphalangeal (PIP) joint is maximally passively flexed, the central slip is normally pulled distally, resulting in slack in the terminal tendon. Injury to the central slip eliminates this slack through the lateral band and allows extensor tension to be generated at the distal interphalangeal (DIP) joint. Thus, with central slip injury, the DIP joint can be actively extended with maximal PIP flexion.

Swan neck deformity results from terminal extensor tendon disruption and total inability to extend the DIP joint independent of PIP position. Flexor digitorum profundus (FDP) avulsion results in inability to flex the DIP joint. In the Elson test, DIP flexion is always possible. PIP volar plate injury may result in jamming of the volar plate within the PIP joint and paradoxical inability to flex (extension contracture) at the PIP joint. There will also be hyperextension PIP joint pain and laxity. Sagittal band disruption results in inability to actively extend at the metacarpophalangeal joint, but the finger can often maintain extension if passively placed in this position.

2014

29
Q

A 3-year-old girl is brought to the office for follow-up because she is unable to flex the interphalangeal joint of the thumb of the dominant right hand. She underwent repair of a laceration of the thenar eminence of the affected hand 8 weeks ago. Operative exploration shows a 3-cm gap of the flexor pollicis longus in Zone III. Reconstruction with a palmaris longus graft is planned. Which of the following is the most appropriate postoperative management?

A) Complete immobilization for 4 weeks
B) Removable dorsal-block splint; passive and active-assist flexion
C) Removable dorsal-block splint; passive flexion
D) Removable dorsal-block splint; passive, active-assist, active flexion
E) No immobilization

A

The correct response is Option A.

Early motion protocols are standard for adult tendon repairs but are not generally suitable for very young children due to poor compliance. Children have a remarkable ability to regain motion after tendon injury, especially for repairs or grafts that are outside of Zone II. In this vignette, the reconstruction was in Zone III, and the prognosis for regaining full motion even after a month of immobilization is excellent. Moreover, it is highly unlikely that a child of this age will predictably comply with splint wear and motion restrictions for the duration of tendon healing. Thus, the risk of early rupture outweighs the risk of stiffness. There are advocates of successful early motion protocols in children with Zone II tendon repairs, but most studies have failed to demonstrate an appreciable benefit of such a practice.

2014

30
Q

A 22-year-old laborer underwent four-strand and epitendinous repair of a Zone II flexor digitorum profundus (FDP) and flexor digitorum superficialis (FDS) injury to the long finger of the dominant right hand 5 days ago. Early active motion therapy protocol is selected for rehabilitation, by which the injured finger is passively flexed and the wrist extended, with the patient then asked to actively maintain a flexed grasp. In contrast to the rubber band Kleinert technique, this protocol is most likely to have which of the following effects on the repaired finger?

A) Decreased risk of tendon rupture
B) Greater risk of finger flexion contracture
C) Increased FDP and FDS excursion
D) More tendon adhesions
E) Prolonged tendon repair softening

A

The correct response is Option C.

Low force and moderate excursion therapy protocols continue to be the most effective protocol following flexor tendon repairs. However, increasing the applied force to the repair site during postoperative rehabilitation beyond 5 N does not accelerate accrual of repair site strength after a multistranded repair. In vivo repair, results have shown that early active mobilization may limit tendon end softening and loss of repair strength that generally occurs after the first 7 days.

The mode of rehabilitation described was popularized by Strickland and has been shown to improve outcomes of Zone II repairs, probably due to both increased absolute as well as relative tendon excursions. However, in order to reduce the potential increased repair rupture rate, at least four-strand repair is required. This increased excursion leads to fewer tendon adhesions.

Because the fingers are flexed by rubber bands, the Kleinert technique predisposes the patient to flexion contractures. A combination of the Duran passive range of motion and the Kleinert technique improves the results.

2014

31
Q

A 42-year-old right-hand–dominant construction worker is evaluated for an 8-week history of pain with wrist motion. Physical examination shows swelling 4 cm proximal to the Lister tubercle. There is tenderness to palpation and crepitation over the muscle bellies of the abductor pollicis longus and extensor pollicis brevis. The remainder of the examination shows no abnormalities. Which of the following is the most likely diagnosis?

A) Basal joint synovitis
B) de Quervain synovitis
C) Extensor pollicis longus tendinitis
D) Intersection syndrome
E) Wartenberg syndrome

A

The correct response is Option D.

Intersection syndrome is synovitis of the second dorsal compartment. It is located where the abductor pollicis longus and extensor pollicis brevis cross the extensor carpi radialis longus and extensor carpi radialis brevis. The syndrome often occurs in athletes with repetitive forceful extension. The swelling is located 4 to 6 cm proximal to Lister tubercle.

Basal joint arthritis and synovitis would have tenderness located at the wrist crease at the carpometacarpal joint and may have x-ray findings. de Quervain is synovitis of the first dorsal compartment, which would be located over the radial styloid and have a positive Finkelstein sign. Wartenberg syndrome is radial sensory nerve compression where the nerve exits the supinator muscle. Symptoms would be paresthesia over the radial nerve distribution and a Tinel sign. Extensor pollicis longus tendinitis is synovitis of the third dorsal compartment with vague dorsal wrist pain and tenderness, usually over the Lister tubercle with exacerbation with thumb extension.

2014

32
Q

A 23-year-old man comes to the office for follow-up evaluation 14 months after sustaining a closed brachial plexus injury in a motor vehicle collision. He was initially treated at another facility with occupational therapy and observation. He has been compliant with therapy. Physical examination shows 4/5 strength in shoulder abduction, elbow flexion, elbow extension, and finger flexion. He is unable to extend the wrist or fingers but has good passive mobility of the wrist and fingers. Which of the following is the most appropriate next step to restore wrist and finger extension?

A) Distal nerve transfer
B) Intraplexus nerve grafting
C) Pedicled latissimus muscle transfer
D) Tendon transfer
E) Continued observation

A

The correct response is Option D.

The most appropriate management for the patient described is tendon transfers to restore wrist, finger, and thumb extension. Following closed brachial plexus injuries, patients should receive CT scan or MR myelogram and electrodiagnostic studies at 3 to 4 weeks. This will allow enough time to see pseudomeningoceles and denervation changes. The electromyography and nerve conduction studies are generally repeated at 14 to 16 weeks to look for signs of regenerating axons, and this information is used to help determine the strategy for reconstruction. Surgery is recommended in the absence of clinical or electrical evidence of recovery.

The patient described has late symptoms, and any strategy that involves attempting to repair or reconstruct the injured nerves is not recommended. After 12 to 18 months, useful motor recovery is unlikely due to intraneural fibrosis, loss of Schwann cells, muscle atrophy, and motor end-plate degeneration. Tendon transfers may be done at any time assuming that there are suitable donor tendons (at least 4/5 strength and full excursion) and that full passive mobility is present. In this example, the patient has adequate donor tendons from the median and ulnar nerves, and full passive range of motion. Examples of tendon transfers include: palmaris longus to extensor pollicis longus, pronator teres to extensor carpi radialis brevis, and flexor carpi ulnaris to extensor digitorum communis.

Early or subacute exploration of the injured plexus with resection and intraplexal nerve grafting can be used with ruptures or neuromas that do not conduct a nerve action potental across the lesion. In adults, grafting is reserved for C5, C6, and C7 to restore shoulder abduction, elbow flexion, elbow extension, and wrist extension. Nerve grafting for lower trunk injuries in adults is generally not successful due to the length and time required for the regenerating axon to reach the distal target muscles.

Nerve transfers have become a very useful and versatile tool for reconstruction of brachial plexus and peripheral nerve injuries. A less important distal nerve is sacrificed to replace the function of a more important nerve. Nerve transfers can be performed in preganglionic injuries and to decrease the distance to the target muscle for reinnervation. Ideally, nerve transfers are performed within 6 months of the injury. Studies have shown greater than 70% will achieve M3 function for elbow flexion and shoulder abduction. Common donor nerves include the spinal accessory, intercostals, anterior interosseous, and triceps branch.

Other options for late reconstruction include pedicled muscle transfers and neurotized functional free muscle transfers. The latissimus dorsi muscle can be used to restore elbow flexion or elbow extension but will not reach beyond the elbow. Currently, free muscle transfer is the best option to restore hand and wrist function in complete brachial plexus palsy.

2014

33
Q

A 24-year-old man with a 1-year history of poorly controlled diabetes mellitus comes to the office 3 months after sustaining a laceration of the left ring finger. Physical examination shows a thick but mobile cutaneous scar. Two-point discrimination in the fingertip is 7 mm, compared with 3 mm in other fingertips. There is no active or passive range of motion in the affected digit. Photographs are shown. Which of the following findings is most likely to preclude reconstruction in this patient?

A) Elevated hemoglobin A1c
B) Hypertrophy of the scar
C) Increased two-point discrimination
D) Length of time from the initial injury
E) Stiffness of the interphalangeal joints

A

The correct response is Option E.

Of the options listed, the most likely option to result in poor outcome in the setting of delayed tendon repair is stiffness of the interphalangeal joints. Hand therapy directed at achieving passive range of motion of the finger before reconstruction may make repair possible and should be attempted before proceeding with surgery.

Although poorly controlled diabetes increases the risk of perioperative infection, it would not impact the ability to perform a reconstruction. Likewise, the amount of time that has passed since the initial injury places the patient outside the range typically accepted for delayed primary repair, but would not directly affect a reconstruction.

Digital nerve injury requiring repair/reconstruction has been considered a relative contraindication to repair. The increased two-point discrimination in this patient suggests a prior injury with recovery similar to what might be expected with primary nerve repair. In this setting, the need for concomitant digital nerve repair is unlikely.

Although thick, the cutaneous scar is mobile, suggesting that it is not contributing to the lack of motion at the interphalangeal joints. Immature scars or wounds requiring further reconstruction would also be contraindications to reconstruction.

2014

34
Q

An otherwise healthy 32-year-old woman returns to the emergency department because she is unable to use her right hand 20 days after she underwent wound closure for management of a laceration to the hand. A photograph of the patient attempting to make a fist is shown. On examination, there is decreased sensation in the ulnar distribution, and the hand is warm. Which of the following tests is likely to provide the most pertinent information in developing a treatment plan for this patient’s injury?

A) CT angiography
B) Electromyography/nerve conduction study
C) Magnetic resonance angiography
D) Ultrasonography
E) No testing is necessary; physical findings are sufficient

A

The correct response is Option E.

The patient described has a significant wrist injury until proven otherwise. Physical examination will be the most helpful in making a determination regarding what should be done next. Electromyography/nerve conduction study typically does not give actionable information until 3 weeks or more after injury. Ultrasonography may be helpful but would not give more information than one could get from a thorough physical examination and would likely be painful as the probe is pressed on the wound. CT angiography can be critical preoperatively if there was a concern regarding inflow. Magnetic resonance angiography would be revealing but would be expensive and unnecessary in this setting.

The patient was taken to the operating room for exploration. Intraoperative and postoperative images are shown.

2014

35
Q

A 30-year-old woman comes to the office because of a laceration of the dorsal long finger extensor tendon. Physical examination shows extension of all fingers to zero degrees at the metacarpophalangeal joint. Which of the following structures provides extension force that explains this exam finding?

A) Deep transverse metacarpal ligament
B) Intact paratenon
C) Interosseous muscles
D) Juncturae tendinum
E) Lumbrical muscles

A

The correct response is Option D.

Juncturae tendinum are tendon-like bands that connect the long, ring, and little finger extensor digitorum communis tendons. If the long finger extensor tendon is lacerated proximal to the attachment of the juncturae tendinum between the long and ring fingers, the ring finger extensor digitorum communis tendon will apply extension force to the distal long finger extensor tendon via the juncturae tendinum and produce metacarpophalangeal (MCP) extension.

Interossesous and lumbrical muscles produce flexion of the MCP joint. The deep transverse metacarpal ligament stabilized the metacarpal heads relative to each other but does not produce motion at the MCP joint. In this patient, the tendon is visible and noted to be completely lacerated; therefore, the paratenon is not intact.

2014

36
Q

An 80-year-old woman comes for evaluation because she is unable to flex the tip of the little finger of her nondominant hand 9 months after sustaining a laceration from a knife. She did not seek treatment at the time of injury. She has no pain or any difficulty with activities of daily living. Physical examination shows a well-healed laceration over the volar aspect of the middle phalanx. Active range of motion is full in the metacarpophalangeal and proximal interphalangeal (PIP) joints; there is no flexion at the distal interphalangeal joint. Which of the following is the most appropriate management?

A) Delayed primary flexor digitorum profundus repair
B) Flexor digitorum superficialis transfer
C) PIP joint arthrodesis
D) Staged flexor tendon reconstruction with placement of a silicone rod followed by tendon grafting
E) No intervention is indicated

A

The correct response is Option E.

No intervention is indicated for this patient. The principles of tendon repair and reconstruction have evolved since the first description of primary tendon repair in Zone II in 1967. Proper patient selection is essential before attempting any reconstruction to restore functional motion.

The indications for grafting or reconstructing through an intact flexor digitorum superficialis (FDS) are narrow, and sacrifice of an intact FDS is generally not recommended. Most of the functional arch of motion is maintained with the superficialis tendon, and many patients will function well with an FDS digit alone. Profundus reconstruction through an intact FDS is most often reserved for patients between 10 and 21 years old with high occupational demands for dexterity such as artists or musicians. This patient is beyond the recommended age range for an optimal outcome and is reporting no functional deficits as a result of her injury. In addition, delayed symptoms may give a clue to the patient’s ability to comply with rigorous postoperative therapy.

Delayed primary repair can be attempted up to several weeks after a flexor tendon injury, and up to 6 weeks in pediatric patients. However, 9 months is well beyond the time when a primary repair would be possible. The FDS from an adjacent finger can be used as the proximal motor in cases of tendon graft reconstruction or tendon repair rupture. This is a consideration if the native proximal stump of the profundus is significantly damaged or scarred and has poor excursion. In this patient, reconstruction is not indicated, and nothing is mentioned regarding the proximal tendon.

A distal interphalangeal (DIP) joint arthrodesis may be indicated if the DIP joint is unstable with a well-functioning proximal interphalangeal (PIP) joint. Tenodesis of the flexor digitorum profundus stump is another option for soft-tissue DIP stabilization.

Tendon reconstruction is indicated when a delay in treatment makes primary tendon repair impossible. A healed wound with full passive range of motion, absence of significant scarring, and an intact flexor retinacular pulley system are considered prerequisites for a single-stage reconstruction. In any other situation, a two-stage reconstruction with implantation of a temporary silicone rod is indicated. This would be the reconstructive strategy of choice for this patient if the FDS tendon were also involved in the original injury.

2014

37
Q

A 50-year-old woman comes to the emergency department after sustaining an avulsion injury of the right ring finger proximal interphalangeal (PIP) joint. A photograph is shown. Examination shows the central slip is disrupted, and the inside of the PIP joint is visible through the dorsal wound. The patient is able to actively extend the PIP joint. Which of the following anatomical structures allows the patient to extend the PIP joint?

A) Extrinsic extensor tendon
B) Interosseous muscle tendon
C) Lateral conjoined tendon
D) Oblique retinacular ligament
E) Sagittal band

A

The correct response is Option B.

The central slip of the extensor mechanism is the terminal direct extension of the extrinsic extensor tendon (extensor digitorum communis and extensor digiti quinti) and is the primary extensor of the proximal interphalangeal (PIP) joint. Injury to the central slip will normally produce flexion of the PIP joint due to unopposed action of the flexor digitorum superficialis (FDS) and flexor digitorum profundus (FDP) tendons and is called a boutonniere deformity.

The intrinsic extensor mechanism, via the middle band of the interosseous muscles, also inserts on the dorsal base of the middle phalanx and causes extension of the PIP joint. In an open injury, the central slip may be injured without concurrent injury to the interosseous muscle tendon, allowing the patient to still actively extend the PIP joint even in the presence of a disruption of the central slip.

The oblique retinacular ligament connects the flexor tendon sheath volarly to the terminal extensor tendon dorsally. When a patient sustains a laceration to the extensor mechanism over the body of the middle phalanx bone, the oblique retinacular ligament may prevent the occurrence of an extensor lag and a mallet deformity.

The lateral conjoined tendon is formed by the lateral bands of the interosseous muscles and the lateral slips of the extrinsic extensor and produces extension of the distal interphalangeal (DIP) joint.

The sagittal band keeps the extrinsic extensor tendon centralized over the dorsal metacarpophalangeal (MP) joint by connecting to the volar plate. Disruption of the sagittal band on one side of a finger would allow the extrinsic extensor tendon to dislocate and impair its ability to extend the MP joint.

2019

38
Q

A 23-year-old man is brought to the emergency department because of a laceration of all extensor tendons at Zone VII of the right upper extremity. Which of the following tendons has the most distal muscle belly when attempting to reappose the tendon ends?

A) Extensor carpi radialis longus
B) Extensor carpi ulnaris
C) Extensor digitorum communis to long finger
D) Extensor indicis proprius
E) Extensor pollicis longus

A

The correct response is Option D.

Zone 7 extensor tendon injuries are those over the dorsal wrist. The extensor indicis proprius tendon typically has the most distal muscle belly and this fact can frequently be used to uniquely identify this tendon. Extensor tendon zones are useful for describing the locations of injuries:

Distal interphalangeal (DIP)

Central slip to DIP

Proximal interphalangeal (PIP)

Metacarpophalangeal (MCP) to PIP

MCP

Carpometacarpal (CMC) to MCP

Wrist and proximal

2019

39
Q

A 50-year-old woman comes to the emergency department because of a stab wound to the right forearm. A photograph is shown. She is taken to the operating room and general anesthesia is administered. Which of the following is the most appropriate examination, in this intubated patient, to assess for tendinous injuries to the fingers?

A) Passively extend the fingers
B) Passively extend the wrist
C) Passively flex the fingers
D) Passively flex the wrist
E) Place hand in cold water

A

The correct response is Option D.

This patient has sustained a dorsal mid forearm laceration. Common injuries within this location include injuries to the musculotendinous units of the extensors to the wrist, fingers, and thumb. Additionally, the radial sensory nerve and dorsal branch of the ulnar nerve could be injured depending on the location and vector of the object that caused the injury. Passive flexion as demonstrated in the photograph demonstrates the effect of tenodesis. With passive wrist flexion, intact digital extensors should be put under tension and bring the metacarpophalangeal joints into extension. However, in this patient the long and ring fingers do not extend with wrist flexion.

In an uninjured hand, when the wrist is passively flexed the fingers and thumb will extend. With wrist extension, the fingers are brought into flexion and the thumb is brought toward the small finger.

Bringing the wrist into extension would help with a volar wound as it would help establish injuries to digital flexors.

Putting the hand in cold water is a better test for sensory nerve injury as loss of wrinkling will be demonstrated.

Passive flexion or extension of the fingers will not reliably demonstrate which specific tendons are injured in this patient.

2019

40
Q

Surgical reconstruction of the oblique retinacular ligament is primarily used to treat which of the following?

A) Boutonnière deformity
B) Flexor digitorum profundus tendon avulsion
C) Sagittal band rupture
D) Scapholunate ligament injury
E) Swan-neck deformity

A

The correct response is Option E.

Spiral oblique retinacular ligament (SORL) reconstruction is used to address digital swan-neck deformity. The procedure is predicated on surgically recreating the proposed function of the oblique retinacular ligament (ORL); linking proximal interphalangeal (PIP) joint and distal interphalangeal (DIP) joint flexion and extension. The ORL arises from the flexor tendon sheath at the base of the proximal phalanx, extends distally and dorsally over the PIP joint, and fuses with the lateral extensor tendon. Because of the site of the ORL origin and insertion, PIP joint extension places the ORL under tension, which results in DIP joint extension. PIP joint flexion, however, decreases tension in the ORL and allows DIP joint flexion. Although anatomical studies are conflicting regarding the actual presence and function of the ORL, surgically recreating this proposed function has been used to treat swan-neck and mallet deformities. Thompson, Littler, and Upton described the dynamic SORL reconstruction in 1978 as an alternative to static techniques, such as transferring a single lateral band volarly to prevent PIP joint hyperextension. In a SORL reconstruction, a free tendon graft is fixated to the dorsal distal phalanx, passed over the radial aspect of the middle phalanx, and then “spiralled” palmarly across the PIP joint, where it is then secured to the ulnar aspect of the proximal phalanx.

A flexor digitorum profundus avulsion (i.e. Jersey finger) is typically managed by reinserting the tendon into the volar base of the distal phalanx. If delayed greater than 8 to12 weeks, interposition tendon reconstruction may be necessary. A boutonniere deformity generally occurs as a result of a central slip injury; this leads to a flexed posture of the PIP joint and an eventual hyperextended posture of the DIP joint as the lateral bands translate volarly. If closed, splinting is indicated. If the central slip was lacerated, an open repair is recommended. A sagittal band tear may cause pain and subluxation of the extensor mechanism at the MCP joint and can be treated with splinting or repair. A scapholunate ligament tear can be directly repaired if acute, or reconstructed using a variety of techniques in the absence of scapholunate advanced collapse and arthritis of the wrist. If arthritis has developed, then partial or total wrist fusion may be indicated.

2021

41
Q

During flexor tendon repair, which of the following is the optimal distance from the cut end of the tendon for placement of core sutures?

A) 1 to 2 mm
B) 3 to 4 mm
C) 7 to 10 mm
D) Greater than 15 mm

A

The correct response is Option C.

The fundamentals of flexor tendon repair are based on primary tendon repair principles, which include easy placement of sutures in the tendon, secure suture knots, smooth juncture of the tendon ends, minimal gapping at the repair site, minimal interference with tendon vascularity, and sufficient strength throughout healing to permit application of early motion stress to the tendon.

These fundamentals are best achieved by incorporating a few basic principles. Handling of the tendon should be minimized to decrease the occurrence of adhesion formation. The strength of the repair is proportional to the number of core sutures and the caliber of the sutures that cross the repair site. The core sutures should be placed 7 to 10 mm from the tendon edge; dorsal placement is biomechanically advantageous.

The distance of the tendon-suture junctions relative to the level of the tendon cut affects the strength of repairs of cut tendons. Strength of repair decreases significantly with purchase distance of less than 4 mm. No increase in strength is seen with purchase distances of greater than 7 mm, and attempts to increase the purchase distance more significantly (greater than 15 mm) will potentially require unnecessary pulley disruption to achieve and will predispose to bunching at the repair site.

2021

42
Q

A 44-year-old woman sustained a second-degree burn to the volar surface of the wrist and palm. She is sent to occupational therapy for fabrication of a splint, placing the wrist and fingers in an intrinsic plus position. Which of the following best describes the position of the wrist and fingers in this splint?

A

The correct response is Option C.

The position of safe immobilization (POSI), also called the intrinsic plus position, was described initially by J.I.P. James (British orthopedic surgeon) and is recognized as the correct position in which to immobilize the hand safely following injury or surgery. The wrist is placed in 0 to 30 degrees of extension, metacarpophalangeal (MCP) joints in 70 to 90 degrees of flexion and interphalangeal (IP) joints in full extension. This position creates pretension on the collateral ligaments of the wrist and the MCP and IP joints of the hand, thereby decreasing the risk of stiffness and contracture.

2021

43
Q

A 56-year-old man is evaluated because of high radial nerve palsy 12 months after sustaining a gunshot wound to the upper arm with complete radial nerve transection. To restore wrist and digit extension, tendon transfers are considered. Which of the following transfers is most appropriate for this patient?

A) Brachioradialis to extensor indicis proprius
B) Flexor carpi radialis to extensor digitorum communis
C) Palmaris longus to extensor pollicis brevis
D) Pronator quadratus to extensor carpi radialis brevis

A

The correct response is Option B.

Tendon transfers for complete high radial nerve injuries are often performed within weeks after injury and allow restoration of wrist and digital extensor stabilization. If present, the palmaris longus tendon is transferred to the extensor pollicis longus tendon to allow for thumb extension. The flexor carpi radialis is transferred to the extensor digitorum communis for finger extension. The pronator teres is transferred to the extensor carpi radialis brevis to add support for wrist extension.

The brachioradialis is not generally a good transfer in a high radial nerve palsy as it is typically weak. The extensor indicis proprius does not usually receive a tendon for transfer as the extensor digitorum communis will provide extension to all digits, including the index. The extensor pollicis brevis does not normally receive a tendon transfer since thumb MP joint extension (in addition to IP joint extension) is normally restored with transfer to the extensor pollicis longus tendon.

Pronator quadratus is not used for tendon transfers for wrist extension and cannot reach the extensor carpi radialis brevis.

2021

44
Q

Under normal conditions, the intrinsic muscles of the hand move the metacarpophalangeal, proximal interphalangeal, and distal interphalangeal joints in which of the following ways?

A

The correct response is Option D.

The intrinsic muscles are those with an origin and insertion within the hand and include the muscles of the thenar and hypothenar compartments, interossei, and lumbricals. They account for approximately 53% of grip strength and 85% of pinch strength. In general, the intrinsic muscles produce metacarpophalangeal joint flexion and interphalangeal joint extension, which occurs because the tendons of lumbrical and interosseous muscles are volar to the metacarpophalangeal axis of rotation but dorsal to the interphalangeal axis of rotation. The exception is when the metacarpophalangeal joint is hyperextended when the dorsal interosseous tendon is capable of producing extension. Tightness of these intrinsic muscles can be evaluated by the Bunnell-Littler test, whereby the force required to passively flex the proximal interphalangeal joint increases with extension of the metacarpophalangeal joint.

2021

45
Q

A 53-year-old woman is evaluated for stiffness of the right index finger. On physical examination, the range of motion of the proximal interphalangeal (PIP) joint is 0 to 45 degrees when the metacarpophalangeal (MCP) joint is in extension (at 0 degrees). With the MCP joint in flexion (at 90 degrees), the PIP joint range of motion is 0 to 80 degrees. Which of the following conditions is most likely to account for these findings?

A) Contracture of the PIP joint capsule
B) Extensor tendon shortening
C) Flexor tendon adhesions
D) Intrinsic tightness
E) Palmar fascia hypertrophy

A

The correct response is Option D.

The condition most likely to account for these examination findings is intrinsic tightness.

This patient exhibits exam signs consistent with stiffness as a result of intrinsic tightness. The Bunnell test for intrinsic tightness involves comparing flexion at the proximal interphalangeal (PIP) joint with the metacarpophalangeal (MCP) joint extended, to flexion at the PIP joint with the MCP joint flexed. In cases of intrinsic tightness, PIP joint flexion will decrease when the MCP joint is extended, and there is increased flexion of the PIP joint when the MCP joint is flexed.

The intrinsic muscles are responsible for flexion at the MCP joints and extension at the PIP joints. When these are tight, PIP joint motion is decreased when the MCP joint is held in extension. There is increased PIP joint flexion possible when the MCP joint is placed in a flexed position because of the decreased tension on the intrinsic muscles that results with MCP flexion, allowing for improved PIP range of motion.

In cases of extensor tendon shortening or tightness (extrinsic extensor tendon contracture), the opposite effect is seen. MCP joint flexion will place additional tension on the extensor mechanism, tightening it and resulting in decreased flexor function at the PIP joint, when compared with an extended MCP joint.

Flexor tendon adhesions and contractures of the PIP joint capsule can result in stiffness but would give equal limitations to PIP flexion regardless of MCP joint position. Palmar fascia hypertrophy is seen in Dupuytren contracture. This contracture could potentially limit PIP extension, which might vary with MCP position, but would not limit flexion of the PIP joint as described in this item.

2021

46
Q

A 57-year-old woman is seen in the emergency department after sustaining a closed crush injury to the dorsum of the right hand. X-ray studies are negative for fracture or dislocation. Examination shows swelling and an inability to actively extend the middle finger at the metacarpophalangeal joint from a flexed position. However, when the finger is placed in extension by the examiner, the patient is able to maintain the finger in extension and resist force. Injury to which of the following structures is the most likely explanation for these findings?

A) Central extensor tendon
B) Juncturae tendinum
C) Lumbrical muscle
D) Oblique retinacular ligament
E) Sagittal band

A

The correct response is Option E.

Injury to the sagittal band is most likely to account for these findings.

The patient has sustained a closed injury to the dorsum of the hand, resulting in a sagittal band rupture. The sagittal bands are responsible for maintaining the position of the extensor tendon dorsally over the metacarpophalangeal (MCP) joint. When this band is ruptured (commonly either via trauma or in rheumatoid arthritis), the extensor tendon can migrate laterally to the MCP joint and fall into the sulcus, causing difficulty in active initiation of extension. However, if the finger is placed in extension passively, the extensor tendon will resume its correct position and will then be able to hold the finger in extension and resist force.

Injury to the central extensor tendon will result in loss of active extension as well as an inability to maintain extension and resist force.

The juncturae tendinum connect the extensor tendons to each other at the dorsum of the hand. Pull via the juncturae may allow for some degree of extension function even in the presence of extensor tendon laceration, but injury to the juncturae should not create a deficit in extension if the tendon remains intact.

The lumbrical muscle is one of the intrinsic hand interosseous muscles. It functions to create flexion at the MCP joint and extension at the interphalangeal joints. Injury to the lumbrical muscle will not cause an extensor deficit at the MCP joint.

The oblique retinacular ligament is in the finger, originating at the base of the proximal phalanx and inserting into the distal phalanx. It is believed to coordinate proximal interphalangeal and distal interphalangeal motion but should have no bearing on motion at the MCP joint.

2021

47
Q

A 30-year-old man who works as an executive presents with a Zone II flexor tendon laceration. He undergoes immediate four-strand core suture flexor tendon repair and epitendinous suture. In the early postoperative period, which of the following approaches is recommended to optimize the outcome in this patient?

A

The correct response is Option B.

Advances in primary flexor tendon repair techniques and postoperative mobilization protocols have improved functional results for Zone II injuries. Repair strength has been shown to be related to the number of strands that cross the repair site. Other factors that contribute to repair strength include suture caliber, adequate core suture purchase (at least 0.7 to 1.0 cm), and the use of an epitendinous repair in addition to core suture placement. A forearm-based dorsal block splint is placed to protect flexor tendon repairs. Compared with passive mobilization protocols, early active mobilization protocols demonstrate better functional outcomes without a significantly increased rate of tendon rupture. A minimum of a four-strand core repair is necessary to tolerate an early active mobilization protocol. In this patient, a six-strand core repair with an epitendinous repair is appropriate for an active motion protocol. Although the flexor repair described in this scenario is strong enough to tolerate protected early active motion, complete absence of a protective splint with unrestricted use of the hand would be inappropriate and risks tendon rupture. The Kleinert splint is a dorsal extension block splint that allows active extension and utilizes rubber bands for passive flexion. This is not considered an early active flexion protocol. A relative motion extension splint with immediate controlled active motion is used following extensor tendon repairs/injuries. Cast immobilization following flexor repair would not be appropriate in this scenario if the goal is to maximize flexor function, but it may have a role in the postoperative management of flexor injuries in children or unreliable adults.

2021

48
Q

A 23-year-old man presents with a deep volar distal forearm laceration after striking a window. He is taken emergently to the operating room for repair of multiple flexor tendon, nerve, and arterial transections. Which of the following best describes the anatomical relationship of the flexor digitorum superficialis tendons so that the proximal and distal stumps can be correctly matched and repaired?

A) Index and middle superficialis tendons are volar to the ring and small superficialis tendons
B) Index and small superficialis tendons are volar to the middle and ring superficialis tendons
C) Index, middle, ring, and small superficialis tendons are arranged side-by-side
D) Middle and ring superficialis tendons are volar to the index and small superficialis tendons
E) Ring and small superficialis tendons are volar to the index and middle superficialis tendons

A

The correct response is Option D.

The superficial location of numerous, tightly packed vital structures at the volar aspect of the wrist/distal forearm makes them susceptible to injury with penetrating trauma. The patient presents with a “spaghetti wrist,” a term used to describe these devastating injuries that may result in transection of multiple flexor tendons and neurovascular structures.

A thorough knowledge of the anatomy in this region is required in order to appropriately align and repair all transected critical structures. In particular, the flexor digitorum superficialis tendons are arranged such that the middle and ring superficialis tendons are superficial to the index and small finger superficialis tendons. Of note, the flexor digitorum profundus tendons are arranged in a side-by-side fashion deep to the superficialis tendons. The median nerve is located between the superficialis and profundus musculotendinous units at this level.

The other options do not describe the correct arrangement of the superficialis tendons at the distal forearm/wrist level.

2021

49
Q

A right-hand–dominant, 72-year-old man presents with clicking, locking, and pain deep to the distal palmar crease of the right ring finger. Four weeks prior he had a 40 mg triamcinolone injection with incomplete resolution of his symptoms. How long should he wait after the corticosteroid injection before performing an A1 pulley release in order to mitigate the increased risk of surgical site occurrence?

A) 1 week from injection
B) 4 weeks from injection
C) 8 weeks from injection
D) 12 weeks from injection
E) There is no increased risk of surgical site occurrence

A

The correct response is Option D.

The risk of surgical site occurrence is mitigated at approximately 80 days (~12 weeks) from corticosteroid injection. In a retrospective review of 999 patients who underwent corticosteroid injection for trigger digit and subsequently underwent surgery, they carefully scrutinized rates of surgical site occurrence. Charts were queried for infection/occurrence by identifying “infection,” “suture abscess,” “worrisome for infection,”, or “return to the operating room for infection.” Those who did not develop an infection had a significantly longer time between corticosteroid injection and surgery (mean 260 days vs mean 79 days, p less than 0.05). There were no differences in infection rates between those who underwent one or multiple corticosteroid injections prior to surgery (Ng et al.).1

In male patients with a single involved digit, the average success rate for corticosteroid injection alone is low (35%). One may suggest that surgery is indicated in this patient population prior to attempting corticosteroid injection. When evaluating the treatment of trigger digit from a cost perspective, males with single digit involvement or multiple digit involvement and women with multiple digit involvement should forgo corticosteroid injection because of low success rates (35%, 37%, and 56%, respectively) (Brozovich et al. and Wojahn et al).2,3

The decision to treat trigger digit with corticosteroid injection versus surgery may also be a personal decision by the patient after informed discussion with the surgeon. It is important to remember that corticosteroid injection preceding surgery may increase the risk for surgical site occurrences.

2021

50
Q

A 25-year-old woman presents with Volkmann contracture. Examination shows the patient has supple fingers but no active flexion of the wrist or digits. MRI shows fibrosis of the volar forearm compartment. Electromyography shows no motor action potentials in the forearm flexor musculature. Which of the following is the most appropriate management of this patient to restore extrinsic finger flexion?

A) Flexor digitorum superficialis to flexor digitorum profundus tendon transfer
B) Flexor pronator slide
C) Free gracilis functioning muscle transfer
D) Pronator teres to extensor carpi radialis brevis tendon transfer
E) Z-lengthening of the flexor tendons in the forearm

A

The correct response is Option C.

This patient has developed Volkmann ischemic contracture, and there has been muscle fibrosis resulting from ischemia of the volar compartment. At this point, there is no functioning flexor muscle. The most appropriate management is thorough debridement of the volar forearm muscles and free functioning muscle transfer to restore extrinsic finger flexion.

Flexor digitorum superficialis to flexor digitorum profundus tendon transfer is a useful treatment to open a nonfunctional hand due to severe spasticity. Pronator teres to extensor carpi radialis brevis tendon transfer is a tendon transfer used to restore wrist extension, not flexion. Flexor pronator slide or lengthening of the flexor tendons is only indicated when there is a contracture but still some functioning muscle. Z-lengthening of the flexor tendons also relies on a mild to moderate contracture and the presence of functioning flexor muscle.

2022

51
Q

A 26-year-old man presents with silicone rods in the flexor tendon sheaths of middle and ring fingers. Medical history includes saw injury resulting in flexor tendon debridement. Second stage flexor tendon reconstruction using an expendable donor tendon graft from the lower leg is planned. Which of the following is the most appropriate description of the location for this tendon at the ankle?

A) Anterior to the lateral malleolus
B) Anterior to the medial malleolus
C) In the anterior compartment
D) In the lateral compartment
E) Lateral to the Achilles tendon
F) Medial to the Achilles tendon

A

The correct response is Option F.

The most appropriate description of the location of the tendon at the ankle is medial to the Achilles tendon.

The plantaris tendon can present an expendable donor tendon from the lower leg for tendon grafting. The plantaris is a small, thin rudimentary muscle with a long tendon. It acts in plantar flexion at the ankle and flexion at the knee, and harvesting does not leave any donor site deficits.

The plantaris originates in the popliteal fossa and travels along the posterior aspect of the calf between the soleus and gastrocnemius muscles to insert in the medial calcaneus. Although there may be some variability in the nature of its insertion, typically the plantaris tendon joins with the Achilles tendon and is found just medial to the Achilles tendon at the ankle in the superficial posterior compartment.

It is present in about 90% of people and is found at a higher incidence than the palmaris longus tendon. The plantaris can provide up to double the length of tendon graft compared with the palmaris longus, and it would be a good choice for reconstructing multiple tendon defects where greater length is needed.

2022

52
Q

Which of the following best describes the origin and insertion of the lumbrical muscles?

A

The correct response is Option A.

The lumbrical muscles are intrinsic muscles of the hand. They arise from the flexor digitorum profundus tendon and insert into the radial lateral band of the extensor mechanism. Their origin and insertion both attach to muscle/tendon rather than bone, which makes them unique compared with other muscles, which typically originate from and insert into bone.

No muscles arise from the flexor digitorum superficialis tendon. The interosseous muscles originate from the metacarpal shafts and insert into the proximal phalanges.

2022

53
Q

A 45-year-old, left-hand–dominant man presents to the emergency department with right thumb pain after a fall while hiking. X-ray studies are shown. Physical examination shows the thumb is swollen, bruised, and tender to palpation. The metacarpophalangeal joint demonstrates laxity of 40 degrees with a firm endpoint on valgus testing. To determine if nonoperative management is an option, which of the following additional tests should be ordered?

A) Arthrography
B) CT scan
C) Electrodiagnostic studies
D) Triple-phase bone scan
E) Ultrasonography

A

The correct response is Option E.

An ultrasound will be the most effective way to diagnose a Stener lesion, which would necessitate surgical intervention for this patient. Stener lesions are a unique type of ulnar collateral ligament injury in which the dorsal adductor aponeurosis becomes interposed between the ruptured distal end of the ligament and its insertion at the proximal phalanx. This prevents healing of the ligament, and thus these injuries cannot be treated closed. Ulnar collateral ligament injuries that demonstrate no endpoint on valgus stress testing generally require operative management. Those that demonstrate some laxity with a firm endpoint can be treated with cast immobilization as long as there is no Stener lesion. Stress view x-ray study will show full versus partial disruption of the ligament but will not identify the presence or absence of a Stener lesion. Some consider MRI to be the most sensitive modality for detection of these lesions; however, criticisms of MRI include its cost and delay in availability. Arthrography can be used to identify capsular injury but cannot detect the collateral ligament displacement with high accuracy. CT scan will not be able to resolve the ligament position clearly. Ultrasonography is cost-effective, dynamic, and easy to obtain. Ultrasonography has been shown to have a positive predictive value of 87 to 100% when used to identify Stener lesions. On the imaging examination, an uninjured collateral ligament will appear as a hypoechoic arc. In a Stener lesion, the arc will be disrupted with displacement or a large gap between the two ends, as seen in the image.

2022

54
Q

A 26-year-old man undergoes acute flexor pollicis longus laceration repair at the interphalangeal (IP) joint level. The proximal stump cannot be retrieved through the laceration. An incision at the distal forearm is performed. After identifying the flexor pollicis longus at this level, an attempt to retrieve the proximal stump is initially prevented by an attachment of the flexor pollicis longus to an adjacent tendon. Which of the following tendons is most likely involved in this anomalous connection?

A) Abductor pollicis longus
B) Brachioradialis
C) Flexor carpi radialis
D) Flexor digitorum profundus index
E) Flexor digitorum superficialis index

A

The correct response is Option D.

An anomalous connection between flexor pollicis longus (FPL) and the profundus tendon to the index finger (flexor digitorum profundus-II) was initially described by Linburg and Comstock in 1979. In their initial report, the Linburg-Comstock anomaly was detected on physical examination in one extremity in 31% of patients and in both extremities in 14%. Dissection of 43 cadavers demonstrated the anomaly in at least one extremity of 25% and in both extremities of 6%. This anomalous tendon connection has been described as a source of tenosynovitis, which is alleviated with surgical division of the connection. The connection typically occurs at the level of the distal forearm, proximal to the carpal tunnel. This connection has been blamed for failure of FPL repair due to transmission of inadvertent tension to the repair site.

2022

55
Q

A 24-year-old man presents with a Leddy and Packer type III avulsion injury to the ring finger flexor digitorum profundus tendon sustained while trying to make a tackle during a pick-up football game. At which of the following levels will the tendon most likely be found during surgical exploration?

A) A4 pulley
B) Carpal tunnel
C) Lumbrical origin
D) Metacarpal head
E) Proximal interphalangeal joint

A

The correct response is Option A.

Avulsion injury of the flexor digitorum profundus (FDP) tendon represents a flexor tendon injury within zone 1 of the flexor tendon sheath. The anatomy of the pulley and vincular systems affects both the level of FDP tendon retraction and the ultimate prognosis. FDP avulsion injuries were classified into three types by Leddy and Packer in 1977. A type I injury describes tendon retraction into the palm, with the tendon tethered by the lumbrical origin. With this injury, both the vinculum profundus longus (VLP) and brevis (VBP) are ruptured. As a result, there is a substantial loss of both the intrinsic (periosteal) and extrinsic (vincular) vascular supply to the tendon. In type II injuries, the FDP tendon retracts to the level of the proximal interphalangeal joint. In this scenario, the VBP is disrupted, but the VLP remains preserved as it arises at the level of the proximal interphalangeal (PIP) volar plate. In type III injuries, retraction to the level of the A4 pulley of the middle phalanx is seen; these injuries are usually defined by an associated large bony fragment incarcerated within the A4 pulley. In direct contrast with type I injuries, both vincula are usually intact in type III injuries because the VBP originates at the distal portion of the middle phalanx. The Leddy and Packer classification has been expanded to include and better classify distinct injury patterns. Type IV injuries are rare and unique in that they include a large avulsion fragment incarcerated at the A4 pulley, along with rupture of the FDP tendon insertion off this osseous fragment, with secondary tendon retraction into the finger or palm. Type V injuries are complex and defined by the presence of concomitant osseous distal phalanx avulsion and distal phalanx fracture.

2022

56
Q

A 25-year-old woman presents with a Zone II laceration of the left index finger flexor digitorum superficialis (FDS) and flexor digitorum profundus (FDP). Repair of the FDP and only one slip of FDS is planned. Compared with repairing both slips of FDS, repair of only one slip of the FDS will result in a decrease of which of the following?

A) Range of motion
B) Rate of tendon healing
C) Resistance
D) Risk for infection
E) Strength of FDP repair

A

The correct response is Option C.

There is a close proximity of the flexor digitorum superficialis (FDS) and flexor digitorum profundus (FDP) tendons in Zone II lacerations. Due to this, adhesion formation is common after injury. Repairing only one slip of the FDS tendon allows for more room within Camper chiasm during motion, which has been found to decrease resistance and improve glide during motion. It has also not been found to limit active range of motion when compared with repairing both FDP and FDS in Zone II. Due to the improvement in glide and decreased resistance, fewer adhesions occur. This also decreases the work of flexion as well. Due to increased adhesion formation with repair of both FDP and FDS in Zone II, there is an increased rate of secondary surgeries for tenolysis. There is no change in the risk for infection or change in the rate of tendon healing with this type of repair compared with repairing both slips of FDS.

2022

57
Q

A 35-year-old woman presents with a laceration of the left middle finger from a kitchen knife. Wound exploration in the emergency department prior to closure demonstrates partial flexor tendon laceration. Which of the following is an absolute indication for operative exploration?

A) Digital artery injury
B) Injury to the A4 pulley
C) 20% Laceration of the flexor digitorum profundus tendon
D) 40% Laceration of the flexor digitorum profundus tendon
E) Triggering

A

The correct response is Option E.

In one large meta-analysis, tendon lacerations up to 90% were managed with early protected range of motion. The only absolute indication for surgery from the list is triggering. There is no evidence that repairing the sheath or pulley without bowstringing is necessary. Isolated A4 laceration would not result in bowstringing. Single digital artery injury in the setting of a perfused digit does not necessitate repair.

2022

58
Q

A 23-year-old man sustains a stab injury to the volar wrist in the region of the carpal tunnel. Examination shows absence of flexion function at the proximal interphalangeal joint of the index finger when the remaining fingers are held in extension. He retains flexor function of the distal interphalangeal joint of the index finger. A diagram of the carpal tunnel contents is shown. Which of the following is the most likely location of the injured tendon?

A

The correct response is Option C.

Finger flexion results from the actions of the flexor digitorum superficialis (FDS) and flexor digitorum profundus (FDP) tendons. The FDS tendon inserts into the middle phalanx and provides for flexion at the proximal interphalangeal (PIP) joint. The FDP tendon inserts into the distal phalanx and is the only tendon that provides flexion at the DIP joints. Because it also crosses the PIP joint, the pull of the FDP tendon can result in flexion at the PIP joint as well. To independently examine the flexor tendons, the FDP tendon can be neutralized by holding the remaining fingers in extension. Due to a common muscle belly, the FDP will not act upon the finger, allowing inspection of the FDS tendon function by evaluating PIP joint flexion in this situation.

The patient has sustained a flexor tendon laceration in the region of the carpal tunnel. On physical examination, there is absence of flexion function at the index finger of the PIP joint when the remaining fingers are held in extension, indicating laceration to the flexor digitorum superficialis of the index finger.

At the level of the carpal tunnel, the flexor tendons travel through a fibro-osseous canal, which contains nine flexor tendons along with the median nerve. The FDS tendons of the middle and ring fingers are most superficial, with the FDS tendons of the index and small fingers deep to them. The FDP tendons lie parallel to each other at the deepest aspects of the carpal tunnel along the bone. The flexor pollicis longus is the most radial structure within the carpal tunnel. The cross-sectional anatomy of the wrist at the level of the carpal tunnel is diagrammed, along with the corresponding tendon locations in the two images shown:

  • A – FDS middle
  • B – FDS ring
  • C – FDS index
  • D – FDS small
  • E – FDP index
  • F – FDP middle

2022

59
Q

A 67-year-old woman underwent cast immobilization for treatment of a minimally displaced left distal radius fracture 5 weeks ago. Two weeks after cast removal, she is unable to extend her thumb. On examination, she is unable to perform retropulsion of the thumb with her palm placed flat on the examination table. Which of the following is the most appropriate next step in management?

A) Electrodiagnostic studies
B) Immobilization
C) Primary tendon repair
D) Tendon transfer
E) X-ray study

A

The correct response is Option D.

This patient is presenting with a rupture of the extensor pollicis longus (EPL) tendon, which occurs in as much as 5% of patients with mildly displaced distal radius fractures about 7 weeks after the initial injury. This is thought to be due to attritional rupture secondary to decreased vascularity. The treatment for this problem is a tendon transfer of the extensor indicis proprius to the EPL. Although some surgeons use a tendon graft, a primary repair is not feasible due to fraying of the EPL. Occupational therapy, immobilization, and further diagnostic studies (including electrodiagnostics and x-ray study) are not appropriate.

2022

60
Q

A 35-year-old man presents with a laceration to the dorsum of his nondominant hand sustained with a kitchen knife. Examination shows a laceration to the metacarpophalangeal (MCP) joint of the long finger. There is no obvious joint involvement, but the patient has an extensor lag of 30 degrees at the MCP joint. Surgical repair of the Zone 5 extensor digitorum communis tendon is performed. Postoperatively, which of the following treatment plans is most likely to provide this patient with the most motion and best outcome?

A) 1 week of immobilization followed by relative motion splint and short arc motion
B) 2 weeks of immobilization followed by dynamic extension splint
C) 3 weeks of immobilization followed by full motion
D) 4 weeks of immobilization followed by protected motion
E) No postoperative orthosis and released to full motion immediately

A

The correct response is Option A.

Traditionally, extensor tendon injuries were treated with 4 to 6 weeks of immobilization that would subsequently lead to decreased flexion and stiffness. More recently, short arc motion protocols with relative motion splints have been utilized and demonstrate improved outcomes. To do this, a thermoplastic yoke splint is constructed by placing the digits of the repaired extensor tendon in approximately 15 to 20 degrees more metacarpophalangeal extension than the adjacent digits for 6 weeks while allowing full interphalangeal joint motion. The wrist is typically placed into an extension splint for 3 weeks at approximately 20 degrees extension to decrease tension on the repair site. Dynamic splinting following extensor tendon repair still limits gliding of the tendon by keeping the digit in a relative static position.

2022